4
$\begingroup$

Hello,

I am reading some material on circle method. Right now I am at its application to the binary Goldbach problem. To obtain a certain bound the fact $\sum_{n> X} 1/ \phi(n)^2 = O(1/X)$ is used. Could anyone please help me how to solve it?

Here $\phi$ is the Euler totient function. And I would like the result without $\epsilon$.

Thank you.

$\endgroup$
2
  • 1
    $\begingroup$ Why do you ask? $\endgroup$
    – Igor Rivin
    Jun 5, 2012 at 16:50
  • $\begingroup$ If I am wrong I apologize but this seems like a homework problem. $\endgroup$ Jun 5, 2012 at 17:23

3 Answers 3

8
$\begingroup$

Theorem 2.14 of "Multiplicative Number Theory" I. Classical Theory, by Montgomery & Vaughan implies that $$\sum_{n\leq x} \left(\frac{n}{\phi(n)}\right)^2 = O(x)$$

Use this and partial summation method with $$\sum_{n\leq x} \left(\frac{n}{\phi(n)}\right)^2 \frac{1}{n^2}$$

$\endgroup$
0
2
$\begingroup$

Or, making different assumptions of what the question means: find a function $\phi$ so that $$ \sum_{n=X}^\infty \frac{1}{\phi(n)^2} = O(1/X)\qquad\text{as } X \to \infty $$ And in fact $\phi(n)=n$ is an example that satisfies that: $$ \sum_{n=X}^\infty \frac{1}{n^2} = \frac{1}{X}+\frac{1}{2X^2}+\frac{1}{6X^3}-\frac{1}{30X^5} + O(1/X^7) $$

$\endgroup$
2
  • $\begingroup$ Whoever down-voted this: Gerald's answer is correct, and it is very hard to tell what the OP is actually asking, as Gerald points out. $\endgroup$
    – Igor Rivin
    Jun 5, 2012 at 20:02
  • $\begingroup$ My apologies for not being clear enough regarding my question. It is my first time on Mathoverflow and I am still trying to figure out how to use this properly... $\endgroup$
    – J Kasahara
    Jun 6, 2012 at 13:45
1
$\begingroup$

From Theorem 7 in Pete Clark's notes it follows that you get $O(1/X^{1-\epsilon})$ for any $\epsilon > 0.$

$\endgroup$
2
  • $\begingroup$ A downvote? Is the answer wrong? $\endgroup$
    – Igor Rivin
    Jun 5, 2012 at 20:02
  • $\begingroup$ Two downvotes. I am honored. $\endgroup$
    – Igor Rivin
    Jun 5, 2012 at 21:12

Your Answer

By clicking “Post Your Answer”, you agree to our terms of service and acknowledge you have read our privacy policy.

Not the answer you're looking for? Browse other questions tagged or ask your own question.